10 Anatomy, Regional Anesthesia, and Pain Management

¡Supera tus tareas y exámenes ahora con Quizwiz!

788.Tachyphylaxis to local anesthetics is MOST closely related to which of the following? A.Speed of injection B.Dosing interval C.Volume of local anesthetic D.pH of solution

788. (B)Tachyphylaxis is a well-known phenomenon associated with repeated injections of local anesthetics leading to decreased effectiveness. Interestingly, the dosing interval seems most important in the development of tachyphylaxis. If the dosing interval is short (and there is no pain between injections), tachyphylaxis does not develop. However, with longer dosing intervals (and pain between injections), tachyphylaxis develops ( Miller: Miller's Anesthesia, ed 8, pp 1051-1052).

796.Which of the following is the MOST important disadvantage of interscalene brachial plexus block compared with other approaches? A.Large volumes of local anesthetics required B.Frequent sparing of the ulnar nerve C.Frequent sparing of the musculocutaneous nerve D.High incidence of pneumothorax

796. (B)The major disadvantage of the interscalene block for hand and forearm surgery is that blockade of the inferior trunk (C8-T1) is often incomplete. Supplementation of the ulnar nerve often is required. The risk of pneumothorax is quite low, but blockade of the ipsilateral phrenic nerve occurs in up to 100% of blocks. This can cause respiratory compromise in patients with significant lung disease. Horner syndrome from blockade of the stellate ganglion can occur in 70% to 90% of patients if large volumes of local anesthetic are injected ( Hebl: Mayo Clinic Atlas of Regional Anesthesia and Ultrasound-Guided Nerve Blockade, ed 1, pp 191-205; Miller: Miller's Anesthesia, ed 8, pp 1724-1727).

789.Which of the following techniques is LEAST effective in the treatment of pruritus from administration of neuraxial opiates? A.Nalbuphine 5 mg IV B.Dexmedetomidine 30 μg IV C.Diphenhydramine 50 mg IV D.Propofol 10 mg IV

789. (B)The treatment of pruritus, the most common side effect of neuraxial opiates, is primarily with opioid antagonists, mixed opioid agonist-antagonists, and antihistamine drugs (by their sedating effects). Nalbuphine is a mixed opioid agonist-antagonist; diphenhydramine has antihistamine properties. Propofol at very low doses (e.g., 10 mg) has been useful to treat pruritus not only induced by neuraxial opiates but also the pruritus associated with cholestatic liver disease. Propofol does not affect analgesia, whereas opioid antagonists and mixed agonist-antagonists may reverse some or all of the analgesia, depending upon dose. Dexmedetomidine is a highly selective α 2-receptor agonist that has a faster onset and shorter duration of action compared with clonidine. Dexmedetomidine has analgesic properties, can potentiate neuraxial analgesia when injected spinally, and can perhaps decrease the incidence of pruritus by reducing the amount of narcotic dose used. It does not treat pruritus ( Barash: Clinical Anesthesia, ed 7, p 519; Miller: Miller's Anesthesia, ed 8, pp 2986-2987).

790.The MAXIMUM dose of lidocaine containing 1:200,000 epinephrine that can be administered to a 70-kg patient for most major regional anesthetic techniques (and excluding spinal and IV regional) is A.100 mg B.200 mg C.500 mg D.1000 mg

790. (C)The maximum dose of local anesthetics containing 1:200,000 epinephrine that can be used for major nerve blocks in a healthy 70-kg adult is lidocaine, 500 mg; mepivacaine, 500 mg; prilocaine, 600 mg; bupivacaine, 225 mg; levobupivacaine, 225 mg; ropivacaine 250 mg ( Miller: Miller's Anesthesia, ed 8, p 1043, Table 36-6).

791.Which of the following concentrations of epinephrine corresponds to a 1:200,000 mixture? A.0.5 μg/mL B.5 μg/mL C.50 μg/mL D.0.5 mg/mL

791. (B)1:200,000 means 1 g/200,000 mL = 1000 mg/200,000 mL = 1 mg/200 mL 1 mg/200 mL = 1000 μg/200 mL = 10 μg/2 mL = 5 μg/mL

792.An anesthesia pain service consult is sought for a 78-year-old patient with a complaint of pain in the distribution of the trigeminal nerve. The patient has no other medical problems except a history of congestive heart failure, for which he takes digoxin and thiazide. In addition to his chief complaint, the patient over the last 72 hours has complained of dysesthesia in the feet, difficulty with vision, and emesis times three. The MOST appropriate step at this time would be A.Trigeminal nerve block with bupivacaine B.Neurologic workup for multiple sclerosis C.Administration of fentanyl and ondansetron D.Obtaining a digoxin level

792. (D)The early signs of digitalis toxicity include loss of appetite and nausea and vomiting. In some patients, there may be pain that is similar to trigeminal neuralgia. Pain or discomfort in the feet and pain and discomfort in the extremities may be a feature of digitalis toxicity. Transient visual disturbances (e.g., amblyopia, scotomata) have been reported in patients with digitalis toxicity. In this patient, it would be prudent to obtain a digoxin level as an early part of the workup for these complaints. He may also have true trigeminal neuralgia, and workup for this condition can be undertaken after digitalis toxicity has been ruled out ( Stoelting: Pharmacology and Physiology in Anesthetic Practice, ed 4, pp 314-315).

793.Which of the following is the EARLIEST sign of lidocaine toxicity from a high blood level? A.Shivering B.Nystagmus C.Light-headedness and dizziness D.Tonic-clonic seizures

793. (C)Toxic reactions to local anesthetics are usually due to intravascular or intrathecal injection or to an excessive dosage. The initial symptoms of local anesthetic toxicity from high blood levels (inadvertent IV injection or excessive dosages) are light-headedness and dizziness, and numbness of the tongue. Patients also may note perioral numbness and tinnitus. Progressive CNS excitatory effects include visual disturbances (difficulty focusing), auditory disturbances (tinnitus), shivering, muscular twitching, and, ultimately, generalized tonic-clonic seizures. CNS depression can ensue, leading to respiratory depression or arrest. Higher levels can lead to cardiovascular collapse. To help prevent excessively high levels of local anesthetic, common practice is to aspirate for blood and inject the local anesthetic slowly and incrementally, looking for signs of toxicity (and, if appropriate, adding epinephrine to use as an intravascular marker as noted by an increase in heart rate and blood pressure) ( Barash: Clinical Anesthesia, ed 7, pp 572-575; Miller: Miller's Anesthesia, ed 8, pp 1048-1052).

794.An analgesic effect similar to the epidural administration of 5 mg of morphine could be achieved by which dose of intrathecal morphine? A.0.05 mg B.0.3 mg C.1 mg D.Morphine should not be injected into the intrathecal space

794. (B)The site of action of spinally administered opiates is the substantia gelatinosa of the spinal cord. Epidural administration is complicated by factors related to dural penetration, absorption in fat, and systemic uptake; therefore, the quantity of intrathecally administered opioid required to achieve effective analgesia is typically much smaller. Lipid-soluble opioids (e.g., fentanyl) have a faster onset of action but a shorter duration of action compared to the more water soluble opioids (e.g., morphine). A dose of 1 to 5 mg of epidural morphine is approximately equal to an intrathecal dose of 0.1 to 0.3 mg of morphine. Onset time for epidural administration is 30 to 60 minutes with a peak effect in 90 to 120 minutes. Onset time for intrathecal administration is shorter than for epidural administration. Duration of 12 to 24 hours of analgesic effect can be expected by either route with morphine ( Barash: Clinical Anesthesia, ed 7, pp 1627-1630; Miller: Miller's Anesthesia, ed 8, pp 2983-2984, Table 98-4).

795.Which local anesthetic undergoes the LEAST hepatic clearance? A.Chloroprocaine B.Bupivacaine C.Ropivacaine D.Lidocaine

795. (A)Commonly injected local anesthetics are divided chemically into two groups: the amino esters (esters) and the amino amides (amides). The esters include procaine, chloroprocaine, and tetracaine (all have one letter i in the name). The amides are lidocaine, mepivacaine, prilocaine, bupivacaine, levobupivacaine, etidocaine, and ropivacaine (all have two i's in the name). The esters undergo plasma clearance by cholinesterases and have relatively short half-lives, whereas the amides undergo hepatic clearance and have longer half-lives ( Butterworth: Morgan & Mikhail's Clinical Anesthesiology, ed 5, pp 266-271; Miller: Miller's Anesthesia, ed 8, p 1046).

847.A 21-year-old patient reports tingling in her thumb during her cesarean section under epidural anesthesia. To which dermatomal level would this correspond? A.C5 B.C6 C.C7 D.C8

847. (B)The thumb corresponds to dermatome C6, the second and middle fingers correspond to dermatome C7, and the fourth and little fingers correspond to dermatome C8 ( Hebl: Mayo Clinic Atlas of Regional Anesthesia and Ultrasound-Guided Nerve Blockade, ed 1, p 86; Miller: Basics of Anesthesia, ed 6, pp 258-260).

797.A 68-year-old woman is to undergo lower extremity surgery under spinal anesthesia. Which of the following statements concerning the immediate physiologic response to the surgical incision is TRUE? A.The cardiovascular (CV) response to stress will be blocked, but the adrenergic response will not B.The adrenergic response to stress will be blocked, but the CV response will not C.Both the adrenergic and CV responses will be blocked D.Neither the adrenergic nor the CV response will be blocked

797. (C)Surgical trauma includes a wide variety of physiologic responses. General anesthesia has no or only a slight inhibitory effect on endocrine and metabolic responses to surgery. Regional anesthesia inhibits the nociceptive signal from reaching the CNS and, therefore, has a significant inhibitory effect on the stress response, including adrenergic, cardiovascular, metabolic, immunologic, and pituitary. This effect is most pronounced with procedures on the lower part of the body and less with major abdominal and thoracic procedures. The variable effect is probably due to unblocked afferents (i.e., vagal, phrenic, or sympathetic) ( Barash: Clinical Anesthesia, ed 7, p 1353; Miller: Miller's Anesthesia ed 8, pp 3139-3141).

798.The "snap" felt just before entering the epidural space represents passage through which ligament? A.Posterior longitudinal ligament B.Ligamentum flavum C.Supraspinous ligament D.Interspinous ligament

798. (B)The structures that are traversed by a needle placed in the midline prior to the epidural space are as follows: skin, subcutaneous tissue, supraspinous ligament, interspinous ligament, and ligamentum flavum. The ligamentum flavum is tough and dense, and a change in the resistance to advancing the needle is often perceived and to many feels like a "snap." The anterior and posterior longitudinal ligaments bind the vertebral bodies together. See also explanation and diagram in Question 870 ( Barash: Clinical Anesthesia, ed 7, pp 913-914; Miller: Miller's Anesthesia, ed 8, pp 1685-1688).

799.The common element thought to be present in cases of cauda equina syndrome after continuous spinal anesthesia is A.Use of microcatheter B.Maldistribution of local anesthetic C.Administration of lidocaine D.Addition of epinephrine

799. (B)The symptoms of cauda equina syndrome include low back pain, bilateral lower extremity weakness, saddle anesthesia, and loss of bowel and bladder control. Pooling of local anesthetics in dependent areas of the spine within the subarachnoid space has been identified as the causative factor in cases of cauda equina syndrome. Microlumen catheters (27-gauge and smaller) may enhance the nonuniform distribution of solutions within the intrathecal space, but cauda equina syndrome has been associated with the use of larger catheters, 5% lidocaine with dextrose, and 2% lidocaine, as well as 0.5% tetracaine ( Barash: Clinical Anesthesia, ed 7, pp 576, 928; Miller: Basics of Anesthesia, ed 6, p 269).

800.When performing a single-shot spinal anesthetic, the level of block for motor, sensory, and sympathetic blocks differs often by at least two dermatomes. Which of the following sequences is correct from the highest to the lowest level of block? A.Sensory, sympathetic, motor B.Sympathetic, sensory, motor C.Sympathetic, motor, sensory D.Sensory, motor, sympathetic

800. (B)Differential nerve blockade is a complex process with both peripheral nerve blocks and central nerve blocks. With spinal anesthesia, the sympathetic nerve block may be anywhere between two and six dermatomes higher than the sensory block, as noted by pin prick. Sensory block is two to three dermatomes higher than the motor block. However, with epidural anesthesia, the sympathetic and sensory blocks tend to be at the same dermatome level and are higher than the motor block ( Barash: Clinical Anesthesia, ed 7, p 923).

801.A 95-year-old woman has persistent and prolonged thoracic pain after a herpes zoster infection. Which of the treatments below would be the LEAST efficacious in the treatment of her pain? A.Oral amitriptyline B.Oral clonidine C.Topical capsaicin ointment D.Topical lidocaine patch

801. (B)Acute herpes zoster is due to the reactivation of the varicella-zoster virus. Acute treatment includes symptomatic pain treatment and antiviral drugs (e.g., acyclovir, famciclovir, or valacyclovir). It is typically a benign and self-limiting disease in patients younger than 50 years of age. As one gets older, the incidence of postherpetic neuralgia (PHN), defined as pain persisting for more than 3 months after resolution of the rash, increases. The incidence of PHN is about 30% to 50% in patients older than 50 years. Treatment of established PHN has been shown to be resistant to interventions and, thus, can be difficult. However, proven therapies include tricyclic antidepressants, anticonvulsants, opioids, topical local anesthetics (e.g., 5% lidocaine patch), topical capsaicin, and TENS. Sympathetic blocks can provide excellent analgesia but are most useful during the more acute stages of the disease rather than during the late chronic stages. Sympathetic blocks in the acute stages may decrease the incidence of PHN. Oral clonidine, which is used to treat hypertension and opioid withdrawal, has not been shown to be an effective treatment for PHN ( Barash: Clinical Anesthesia. ed 7, p 1657; Butterworth: Morgan & Mikhail's Clinical Anesthesiology, ed 5, pp 1049-1050; Raj: Practical Management of Pain, ed 3, pp 187-189).

802.The deep peroneal nerve innervates the A.Lateral aspect of the dorsum of the foot B.Entire dorsum of the foot C.Web space between the great toe and the second toe D.Medial aspect of the dorsum of the foot

802. (C)The deep peroneal nerve innervates the short extensors of the toes and the skin of the web space between the great and second toe. The deep peroneal nerve is blocked at the ankle by infiltration between the tendons of the anterior tibial and extensor hallucis longus muscles ( Hebl: Mayo Clinic Atlas of Regional Anesthesia and Ultrasound-Guided Nerve Blockade, ed 1, pp 424-427, 446-450).

803.The correct arrangement of local anesthetics in order of their ability to produce cardiotoxicity from most to least is A.Bupivacaine, lidocaine, ropivacaine B.Bupivacaine, ropivacaine, lidocaine C.Ropivacaine, bupivacaine, lidocaine D.Lidocaine, ropivacaine, bupivacaine

803. (B)CNS toxicity from local anesthetics generally parallels anesthetic potency (e.g., bupivacaine is four times as potent as lidocaine, and ropivacaine is three times as potent as lidocaine). Cardiovascular (CV) toxicity occurs at a higher blood level than CNS toxicity. For bupivacaine and ropivacaine, CV toxicity occurs at two times the CNS dose, whereas for lidocaine the CV toxicity occurs at seven times the CNS toxicity levels, making lidocaine the least cardiotoxic and bupivacaine the most cardiotoxic of the listed local anesthetics ( Barash: Clinical Anesthesia, ed 7, pp 573-575; Miller: Miller's Anesthesia, ed 8, pp 1049-1050).

804.Allodynia is defined as A.Spontaneous pain in an area or region that is anesthetic B.Pain initiated or caused by a primary lesion or dysfunction in the nervous system C.An increased response to a stimulus that is normally painful D.Pain caused by a stimulus that does not normally provoke pain

804. (D)The International Association for the Study of Pain (IASP) has defined several pain terms. Anesthesia dolorosa refers to spontaneous pain in an area or region that is anesthetic. Neuropathic pain is pain initiated or caused by a primary lesion or dysfunction in the nervous system. Dysesthesia is an unpleasant abnormal sensation, whether spontaneous or evoked. Hyperalgesia is an increased response to a stimulus that is normally painful. Allodynia is pain caused by a stimulus that does not normally provoke pain ( Barash: Clinical Anesthesia, ed 7, pp 1649-1650; Butterworth: Morgan & Mikhail's Clinical Anesthesiology, ed 5, pp 1025-1026).

805.The primary mechanism by which the action of tetracaine is terminated when used for spinal anesthesia is A.Systemic absorption B.Uptake into neurons C.Hydrolysis by pseudocholinesterase D.Hydrolysis by nonspecific esterases

805. (A)Ester local anesthetics are hydrolyzed by cholinesterase enzymes that are present mainly in plasma and, in a smaller amount, in the liver. Because there are no cholinesterase enzymes present in cerebrospinal fluid (CSF), the anesthetic effect of tetracaine will persist until it is absorbed into systemic circulation. The rate of hydrolysis varies, with chloroprocaine being fastest, procaine intermediate, and tetracaine the slowest. Toxicity is inversely related to the rate of hydrolysis; tetracaine is, therefore, the most toxic of the three esters listed in this question ( Butterworth: Morgan & Mikhail's Clinical Anesthesiology, ed 5, pp 270-271).

806.Complex regional pain syndrome type I (reflex sympathetic dystrophy [RSD]) is differentiated from complex regional pain syndrome type II (causalgia) by knowledge of its A.Etiology B.Chronicity C.Type of symptoms D.Rapidity of onset

806. (A)Complex regional pain syndrome type I (CRPS type I), also called RSD, is a clinical syndrome of continuous burning pain, usually occurring after minor trauma. Patients present with various sensory, motor, autonomic, and trophic changes. Complex regional pain syndrome type II (CRPS type II; causalgia) exhibits the same features of RSD, but there is a preceding nerve injury (e.g., median nerve of the upper extremity or tibial division of the sciatic nerve in the lower extremity) ( Barash: Clinical Anesthesia, ed 7, pp 1657-1658; Butterworth: Morgan & Mikhail's Clinical Anesthesiology, ed 5, pp 1048-1049).

807.The primary determinant of local anesthetic potency is A.pKa B.Molecular weight C.Lipid solubility D.Protein binding

807. (C)The potency of local anesthetics is directly related to their lipid solubility. In general, the speed or onset of action of local anesthetics is related to the pKa of the drug. Drugs with lower pKa values have a higher amount of non-ionized molecules at physiologic pH and penetrate the lipid portion of nerves faster (an exception is chloroprocaine, which has a fast onset of action that may be related to the higher concentration of drug used) ( Barash: Clinical Anesthesia, ed 7, pp 566-567; Butterworth: Morgan & Mikhail's Clinical Anesthesiology, ed 5, pp 268-269).

808.Which of the following would have the GREATEST effect on the level of sensory blockade after a subarachnoid injection of hyperbaric 0.75% bupivacaine? A.Patient age B.Addition of epinephrine to the local anesthetic solution C.Patient weight D.Patient position

808. (D)Many factors have an effect on the sensory level after a subarachnoid injection. The baricity of the solution and the patient position (e.g., lateral, sitting, prone) are the most important determinants of sensory level. The other listed options have little to no effect on sensory level. Patient height also has little effect on sensory level ( Barash: Clinical Anesthesia, ed 7, pp 916-919; Miller: Miller's Anesthesia, ed 8, pp 1693-1694).

809.Which of the following local anesthetics would produce the LOWEST concentration in the fetus relative to the maternal serum concentration during a continuous lumbar epidural? A.Ropivacaine B.Bupivacaine C.Lidocaine D.Chloroprocaine

809. (D)Chloroprocaine is an ester local anesthetic that is rapidly metabolized by pseudocholinesterase. With the epidural injection of chloroprocaine, very little drug is available to cross the placenta, because the half-life is about 45 seconds in the mother (and that which crosses is also rapidly metabolized, making fetal effects essentially nonsignificant). The amide local anesthetics (e.g., ropivacaine, bupivacaine, lidocaine) undergo liver metabolism and have relatively long half-lives, but with prolonged epidural administration may accumulate in the fetus ( Barash: Clinical Anesthesia, ed 7, p 1148; Miller: Miller's Anesthesia, ed 8, p 2344).

810.Severe hypotension associated with high spinal anesthesia is caused primarily by A.Decreased cardiac output secondary to decreased preload B.Decreased systemic vascular resistance C.Decreased cardiac output secondary to bradycardia D.Decreased cardiac output secondary to decreased myocardial contractility

810. (A)Hypotension with a high spinal anesthesia is related to sympathetic blockade, venodilation (decreases preload), arterial dilation (decreases afterload), and a decrease in heart rate (cardioaccelerator fibers T1-T4 blockade and a fall in right atrial filling that affects the intrinsic chronotropic stretch receptors). With a high spinal, the decrease in venous dilation is the predominant cause of hypotension ( Barash: Clinical Anesthesia, ed 7, pp 923-925; Miller: Miller's Anesthesia, ed 8, pp 1688-1690; Miller: Basics of Anesthesia, ed 6, p 270).

811.Select the one TRUE statement concerning phantom limb pain. A.The incidence of phantom limb pain increases with more distal amputations B.Most amputees do not experience phantom limb pain C.Nerve blocks may be used to decrease the incidence of phantom limb pain D.Traumatic amputees have a much higher incidence of phantom limb pain than nontraumatic amputees

811. (C)The incidence of phantom limb pain is estimated to be up to 80% after an amputation. This pain may be immediate but, in many cases, will develop within a few days of the amputation. The pain also may not be present all the time but only a few days a month. The incidence of phantom limb pain does not differ between traumatic and nontraumatic amputees. The incidence of phantom pain increases with more proximal amputation. About 50% of patients will have a decrease in pain over time; the rest have no change or an increase in pain with time. Although very difficult to treat, nerve blocks are commonly used in the perioperative setting to decrease the incidence of phantom limb pain. Oral agents such as opioids, antidepressants, and gabapentin are commonly used as well as TENS units, spinal cord stimulators, and biofeedback methods ( Barash: Clinical Anesthesia, ed 7, p 1658).

812.Which of the following is TRUE regarding intravenous regional anesthesia (Bier block)? A.Useful for postoperative pain in extremity surgery B.Can be used for extremity surgeries lasting 2 to 3 hours C.Bupivacaine is the drug of choice for prolonged blocks D.Lidocaine is most commonly used

812. (D)Intravenous regional anesthesia (IVRA, or Bier blocks after August Bier, who first described the technique) is simple to perform and is usually done only on an upper extremity. A small 20- or 22-gauge IV catheter is placed in the extremity to be blocked, then the limb is raised and an Esmarch bandage is wrapped around the extremity to remove as much blood from the limb as possible, followed by the inflation of a tourniquet to 250 to 300 mm Hg, or 2.5 times the patient's systolic pressure, and injection of a local anesthetic into the limb. An intravenous line is always placed in another site (not below the tourniquet) in case sedation is needed for tourniquet pain or if local anesthetic toxicity develops when the tourniquet is eventually released. Typically, a minimum of 40 to 45 minutes of tourniquet time is needed to have enough local anesthetic to diffuse into the tissues to prevent serious systemic local anesthetic toxicity from developing when the tourniquet is deflated. For safety, the tourniquet is deflated for about 5 seconds and then reinflated for 45 seconds while one looks for signs of toxicity. This should be repeated four to five more times. Postoperative analgesia is lost once the tourniquet is deflated and the local anesthetic diffuses from the nerves. Tourniquet times less than 60 to 90 minutes are used to prevent pain and nerve damage from the tourniquet. Lidocaine 0.5% at a dose of 1.5 to 3 mg/kg is the most commonly administered local anesthetic because of its relative safety and effectiveness. About a 10-minute period is needed for surgical anesthesia to develop. Bupivacaine is not recommended for Bier blocks because of reports of cardiovascular toxicity and death that have occurred after the tourniquet was released ( Barash: Clinical Anesthesia, ed 7, p 970; Hebl: Mayo Clinic Atlas of Regional Anesthesia and Ultrasound-Guided Nerve Blockade, pp 317-320; Miller: Basics of Anesthesia, ed 6, pp 194, 297).

813.Select the FALSE statement regarding spinal anatomy and spinal anesthesia. A.The addition of phenylephrine to lidocaine will prolong spinal anesthesia B.A high thoracic sensory block will result in total sympathetic blockade C.The largest vertebral interspace is L5-S1 D.The dural sac extends to the S4-S5 interspace

813. (D)Both phenylephrine and epinephrine will prolong a spinal anesthetic when administering lidocaine. The Taylor approach for spinal anesthesia uses a paramedian approach to the L5-S1 interspace—the largest interspace of the vertebral column. The sympathetic nervous system originates in the thoracic and lumbar spinal cord T1-L3; therefore, a high thoracic sensory level can cause a complete sympathetic block. The dural sac extends to S2, not S4-S5. The spinal cord extends to L3 in the infant and L1-L2 in adults ( Barash: Clinical Anesthesia, ed 7, pp 906-920; Miller: Miller's Anesthesia, ed 8, pp 1684-1693).

814.Four days after a left total hip arthroplasty, an obese 62-year-old woman complains of severe back pain in the region where the epidural was placed. Over the ensuing 72 hours, the back pain gradually worsens and a severe aching pain that radiates down the left leg to the knee develops. The MOST likely diagnosis is A.Epidural abscess B.Epidural hematoma C.Anterior spinal artery syndrome D.Meralgia paresthetica

814. (A)Development of an epidural abscess is fortunately an exceedingly rare complication of spinal and epidural anesthesia. Most anesthetic-related epidural abscesses are associated with epidural catheters. When an epidural abscess is developing, prompt recognition and treatment are essential if permanent sequelae are to be avoided. Symptoms from an epidural abscess may not become apparent until several days (mean, 5 days) after placement of the block. There are four clinical stages of epidural abscess symptom progression. Initially, localized back pain develops. The second stage includes nerve root or radicular pain. The third stage involves motor and sensory deficits or sphincter dysfunction, followed by the last stage of paraplegia. Unlike an epidural hematoma, in which severe back pain is the key feature, patients with epidural abscesses will complain of radicular pain approximately 3 days after development of the back pain. Fever may develop with an abscess and is rare with a hematoma. A magnetic resonance imaging (MRI) scan is helpful in the diagnosis. Anterior spinal artery syndrome is characterized predominantly by motor weakness or paralysis of the lower extremities. Meralgia paresthetica is related to entrapment of the lateral femoral cutaneous nerve as it courses below the inguinal ligament and is associated with burning pain over the lateral aspect of the thigh. It is not a complication of epidural anesthesia ( Butterworth: Morgan & Mikhail's Clinical Anesthesiology, ed 5, pp 970-972).

815.Which of the following choices is NOT consistent with a limb affected by complex regional pain syndrome? A.Allodynia B.Dermatomal distribution of pain C.Atrophy of the involved extremity D.Hyperesthesia

815. (B)Complex regional pain syndromes are associated with trauma. The main feature is burning and continuous pain that is exacerbated by normal movement, cutaneous stimulation, or stress, usually weeks after the injury. The pain is not anatomically distributed. Other associated features include cool, red, clammy skin and hair loss in the involved extremity. Chronic cases may be associated with atrophy and osteoporosis ( Barash: Clinical Anesthesia, ed 7, pp 1657-1658; Butterworth: Morgan & Mikhail's Clinical Anesthesiology, ed 5, pp 1048-1049).

816.The MAIN advantage of neurolytic nerve blockade with phenol versus alcohol is A.Denser blockade B.Blockade is permanent C.The effects of the block can be evaluated immediately D.The block is less painful

816. (D)Neurolytic blockade with phenol (6%-10% in glycerine) is painless because phenol has a dual action as both a local anesthetic and a neurolytic agent. The initial block wears off over a 24-hour period, during which time neurolysis occurs. For this reason you must wait a day to determine the effectiveness of the neurolytic block. Alcohol (50%-100% ethanol) is painful on injection and should be preceded by local anesthetic injection. Unfortunately, there is no neurolytic agent that affects only sympathetic fibers ( Barash: Clinical Anesthesia, ed 7, pp 1658-1659; Miller: Miller's Anesthesia, ed 8, pp 1910-1911).

817.How much local anesthetic should be administered per spinal segment to patients between 20 and 40 years of age receiving a lumbar epidural anesthetic? A.0.25 to 0.5 mL B.0.5 to 1.0 mL C.1 to 2 mL D.2 to 3 mL

817. (C)In general, each 1 to 2 mL of local anesthetic will anesthetize about one spinal segment in the 20- to 40-year-old patient. Because of the negative intrathoracic pressure transmitted to the epidural space with breathing, about two thirds of the segments are blocked above the level of the lumbar placement and one third of segments are blocked below the injection. For example, to achieve a T4 block when an epidural is placed at the L2-L3 space, about 10 segments above and five segments below the epidural would be needed (15 segments) or about 15 to 30 mL. As one gets older, the dose of local anesthetic mL/segment decreases (e.g., an 80-year-old may need 0.75-1.5 mL/segment). Also, pregnant patients are more sensitive to local anesthetics, and reduced doses are needed ( Barash: Clinical Anesthesia, ed 7, pp 920-922; Butterworth: Morgan & Mikhail's Clinical Anesthesiology, ed 5, p 962; Miller: Basics of Anesthesia, ed 6, p 277).

818.The artery of Adamkiewicz MOST frequently arises from the aorta at which spinal level? A.T1-T4 B.T5-T8 C.T9-T12 D.L1-L4

818. (C)Blood supply to the spinal cord comes from several sources. The anterior spinal artery is derived from the vertebral arteries and runs the entire length of the spinal cord and supplies the anterior two thirds of the cord. There are segmental arteries from the aorta that join the anterior spinal artery to help supply the spinal cord. One of the larger arteries is called the artery of Adamkiewicz, which arises from the lower thoracic area (T9-T12). Damage to this artery can lead to ischemia for the lower two thirds of the spinal cord and paraplegia. The posterior one third of the cord is supplied by two posterior spinal arteries that also arise from the vertebral arteries and receive some blood supply from the segmental arteries ( Barash: Clinical Anesthesia, ed 7, pp 997-998; Miller: Basics of Anesthesia, ed 6, pp 260-261).

819.Which local anesthetic has the longest elimination half-time (T ½)? A.Bupivacaine B.Lidocaine C.Mepivacaine D.Ropivacaine

819. (A)Amino ester local anesthetics undergo hydrolysis in the bloodstream and tend to have short elimination half-times. Amino amides undergo biotransformation by the liver and have longer elimination half-times. The elimination half-time for bupivacaine is 3.5 hours, for levobupivacaine is 3.5 hours, for lidocaine is 1.6 hours, for mepivacaine is 1.0 hour, for procaine is 0.1 hour, and for ropivacaine is 1.9 hours ( Hemmings: Pharmacology and Physiology for Anesthesia, ed 1, p 298).

820.Important landmarks for performing a sciatic nerve block (classic approach of Labat) include A.Iliac crest, sacral hiatus, and greater trochanter B.Iliac crest, coccyx, and greater trochanter C.Posterior superior iliac spine, coccyx, and greater trochanter D.Posterior superior iliac spine, greater trochanter, and sacral hiatus

820. (D)To perform a sciatic nerve block, first draw a line from the posterior superior iliac spine to the greater trochanter of the femur, then draw a 5-cm line perpendicular from the midpoint of this line caudally and a second line from the sacral hiatus to the greater trochanter. The intersection of the second line with the perpendicular line marks the point of entry ( Hebl: Mayo Clinic Atlas of Regional Anesthesia and Ultrasound-Guided Nerve Blockade, ed 1, pp 405-412; Miller: Miller's Anesthesia, ed 8, pp 1742-1743).

821.A 76-year-old female patient is undergoing a carotid endarterectomy under a deep cervical plexus nerve block. Which of the following complications would be LEAST likely with this unilateral block? A.Unilateral phrenic nerve paralysis B.Subarachnoid injection C.Blockade of the spinal accessory nerve D.Vertebral artery injection

821. (C)Deep cervical plexus blocks (C2, C3, and C4) can be used for unilateral neck anesthesia for carotid endarterectomy and cervical node dissections. Complications of deep cervical plexus block include injection of the local anesthetic into the vertebral artery, subarachnoid space, or epidural space. Other nerves that may be anesthetized include the phrenic nerve (which is why bilateral deep cervical plexus blocks should be performed with caution, if at all), and the recurrent laryngeal nerve. Some local anesthetic may spread outside the deep cervical fascia and may produce blockade of the sympathetic chain, producing Horner syndrome. Inadvertent blockade of the recurrent laryngeal nerve has also been reported. The spinal accessory nerve is cranial nerve XI and innervates the sternocleidomastoid muscle as well as the trapezius muscle. The accessory nerve comes out cephalad to the injections ( Barash: Clinical Anesthesia, ed 7, pp 946-947; Hebl: Mayo Clinic Atlas of Regional Anesthesia and Ultrasound-Guided Nerve Blockade, ed 1, pp 179-185).

822.A retrobulbar block anesthetizes each of the following nerves EXCEPT A.Ciliary nerves B.Cranial nerve III (oculomotor nerve) C.Cranial nerve V (facial nerve) D.Cranial nerve VI (abducens nerve)

822. (C)A retrobulbar block anesthetizes the three cranial nerves responsible for movement of the eye (cranial nerve III—oculomotor nerve, cranial nerve IV—trochlear nerve, and cranial nerve VI—abducens nerve). The ciliary ganglion (deep within the orbit and lateral to the optic nerve) and ciliary nerves are also blocked, providing anesthesia to the conjunctiva, cornea, and uvea. Branches of the facial nerve (cranial nerve V) are not blocked by the retrobulbar block but are often separately blocked to produce akinesia of the eyelids ( Barash: Clinical Anesthesia, ed 7, pp 1383-1386; Brown: Atlas of Regional Anesthesia, ed 3, pp 185-188).

823.Which of the following muscles of the larynx is innervated by the external branch of the superior laryngeal nerve? A.Vocalis muscle B.Thyroarytenoid muscles C.Posterior cricoarytenoid muscle D.Cricothyroid muscle

823. (D)The vagus nerve innervates the airway by two branches: the superior laryngeal nerves and the recurrent laryngeal nerves. All of the muscles of the larynx are innervated by the recurrent laryngeal nerve except for the cricothyroid muscle. The superior laryngeal nerve divides into the internal and external laryngeal branches. The external laryngeal branch innervates the cricothyroid muscle. The internal laryngeal branch provides sensory fibers to the cords, epiglottis, and arytenoids ( Barash: Clinical Anesthesia, ed 7, pp 763-764; Butterworth: Morgan & Mikhail's Clinical Anesthesiology, ed 5, pp 310-312).

824.All the following agents are acceptable for use in a Bier block EXCEPT A.0.5% Lidocaine B.0.5% Mepivacaine C.0.25% Bupivacaine D.0.5% Prilocaine

824. (C)Because of the potential for cardiotoxicity and because bupivacaine has no advantages over other local anesthetics in this setting, it is contraindicated for use in intravenous regional anesthesia ( Miller: Miller's Anesthesia, ed 8, p 1736).

825.The stellate ganglion lies in closest proximity to which of the following vascular structures? A.Common carotid artery B.Internal carotid artery C.Vertebral artery D.Aorta

825. (C)The stellate ganglion usually lies in front of the neck of the first rib. The vertebral artery lies anterior to the ganglion, as it has just originated from the subclavian artery. After passing over the ganglion, it enters the vertebral foramen and lies posterior to the anterior tubercle of C6 ( Brown: Atlas of Regional Anesthesia, ed 3, pp 199-203; Miller: Miller's Anesthesia, ed 8, p 1732).

826.Which of the following structures in the antecubital fossa is the MOST medial? A.Brachial artery B.Radial nerve C.Tendon of the biceps D.Median nerve

826. (D)The median nerve is the most medial structure in the antecubital fossa. To block this nerve, first the brachial artery is palpated at the level of the intercondylar line between the medial and lateral epicondyles, and then a needle is inserted just medial to the artery and directed perpendicularly to the skin ( Butterworth: Morgan & Mikhail's Clinical Anesthesiology, ed 5, pp 994-995; Hebl: Mayo Clinic Atlas of Regional Anesthesia and Ultrasound-Guided Nerve Blockade, ed 1, pp 286-288).

827.During placement of an epidural in a 78-year-old patient scheduled for a total knee arthroplasty, the patient complains of a sharp sustained pain radiating down his left leg as the catheter is inserted to 2 cm. The MOST appropriate action at this time would be to A.Leave the catheter at 2 cm, and give a test dose B.Give a small dose to relieve pain, then advance 1 cm C.Withdraw the catheter 1 cm, then give a test dose D.Withdraw the needle and catheter, then reinsert in a new position

827. (D)When an epidural catheter is placed without fluoroscopic guidance, the exact location of the needle tip relative to the anatomic structures of the back can only be surmised. If malposition of either the needle or the catheter is suspected, it is prudent to withdraw the entire apparatus and reinsert a second time. In this case, it is possible that the catheter tip has found its way into a nerve root. Under these circumstances, injection of a local anesthetic or narcotic could produce pressure that could possibly lead to ischemia and neurologic damage. During placement or injection of a needle or epidural catheter, a paresthesia that is sustained is always a warning sign that should be heeded ( Barash: Clinical Anesthesia, ed 7, p 910; Butterworth: Morgan & Mikhail's Clinical Anesthesiology, ed 5, p 949; Raj: Practical Management of Pain, ed 3, p 650).

828.Cutaneous innervation of the plantar surface of the foot is provided by the A.Sural nerve B.Posterior tibial nerve C.Saphenous nerve D.Deep peroneal nerve

828. (B)There are five nerves that innervate the ankle and foot: the posterior tibial, sural, superficial peroneal, deep peroneal, and saphenous nerves. These nerves are superficial at the level of the ankle and are easy to block. The posterior branch of the tibial nerve gives rise to the medial and lateral plantar nerves, which supply the plantar surface of the foot ( Barash: Clinical Anesthesia, ed 7, pp 990-991; Hebl: Mayo Clinic Atlas of Regional Anesthesia and Ultrasound-Guided Nerve Blockade, ed 1, pp 444-448).

829.Which of the following local anesthetics has the LOWEST ratio of dosage required for cardiovascular collapse to dosage required for central nervous system (CNS) toxicity? A.Lidocaine B.Etidocaine C.Bupivacaine D.Prilocaine

829. (C)In general, in both in vivo and in vitro studies there is an overall direct correlation between anesthetic's potency and its direct depressant effect on myocardial contractility. The ratio of dosage required for cardiovascular system (CVS) toxicity in animal models compared with CNS toxicity is lowest for bupivacaine, levobupivacaine, and ropivacaine (2.0). Ratios for other local anesthetics are as follows: prilocaine, 3.1; procaine and chloroprocaine, 3.7; etidocaine, 4.4; lidocaine and mepivacaine, 7.1. Remember that this question refers only to the ratio of CVS to CNS toxicity; it does not refer to which drug is more cardiotoxic ( Barash: Clinical Anesthesia, ed 7, pp 572-575).

830.A 57-year-old patient is scheduled for hemorrhoidectomy. The patient has a history of mild chronic obstructive pulmonary disease, hypertension, and traumatic foot amputation from a tractor accident. His only hospitalizations were for two suicide attempts related to phantom limb sensations 10 years ago. He takes phenelzine (Nardil), thiazide, and potassium. Which of the following anesthetic techniques would be MOST appropriate for this patient? A.Spinal anesthetic with 0.5% hyperbaric bupivacaine B.Epidural anesthetic with 0.5% bupivacaine C.Local infiltration with lidocaine and epinephrine, sedation with propofol and meperidine D.General anesthesia with propofol, succinylcholine, nitrous oxide, and fentanyl

830. (D)Reactivation of phantom limb sensations has been reported in patients who have received both spinal and epidural anesthetics (90% in some series). In the majority of these cases (80%), phantom limb sensation persisted until the block receded. With a history of phantom limb sensations that drove this patient to attempt suicide, it is probably wise to avoid spinal and epidural anesthetics. Phenelzine (Nardil) is a monoamine oxidase (MAO) inhibitor that is occasionally used for the treatment of depression. Any anesthetic or combination of techniques that involves meperidine is contraindicated in patients receiving MAO inhibitors. The combination of meperidine and MAO inhibitors has been associated with hyperthermia, hypotension, hypertension, ventilatory depression, skeletal muscle rigidity, seizures, and coma. Because of this unfavorable drug interaction, meperidine should be avoided in patients receiving MAO inhibitors. Accordingly, the only acceptable choice in this question would be general anesthesia with propofol, succinylcholine, nitrous oxide, and fentanyl. As an interesting side point, the drug phenelzine prolongs the duration of action of succinylcholine by decreasing plasma cholinesterase activity ( Miller: Miller's Anesthesia, ed 8, p 909; Raj: Practical Management of Pain, ed 3, p 212; Waldman: Pain Management, ed 2, Chapter 32).

831.If the recurrent laryngeal nerve were transected bilaterally, the vocal cords would A.Be in the open position B.Be in the closed position C.Be in the intermediate position (i.e., 2-3 mm apart) D.Not be affected unless the superior laryngeal nerve were also injured

831. (C)The recurrent laryngeal nerve innervates all the muscles of the larynx (e.g., abductors and adductors) except the cricothyroid muscle (which tenses the vocal cords and is innervated by the external branch of the superior laryngeal nerve). With complete bilateral transections of the recurrent laryngeal nerve, both the abductor and adductor muscles are affected, and the vocal cords will adopt an intermediate position (i.e., lie within 2-3 mm of the midline). Acute complete injury to the recurrent laryngeal nerves can result in stridor and respiratory distress requiring treatment (e.g., intubation and possible tracheostomy). If a patient sustained a partial bilateral paralysis of the recurrent laryngeal nerve that affected only the abductor muscles, then the unopposed adductor muscles would bring the cords together (i.e., closed) and complete airway obstruction would ensue ( Butterworth: Morgan & Mikhail's Clinical Anesthesiology, ed 5, pp 310-312; Miller: Miller's Anesthesia, ed 8, p 2526).

832.A 63-year-old woman undergoes total knee arthroplasty under spinal anesthesia. Two days later she complains of a severe headache. Pain intensity is not related to posture. The LEAST likely cause of this headache is A.Caffeine withdrawal B.Viral illness C.Migraine D.Postdural puncture headache (PDPH)

832. (D)PDPH is due to a loss of CSF through a dural puncture and characteristically has a postural component. When supine, the headache is usually gone but may be mild in some cases. When the head is elevated, the headache may be severe, is bilateral, and may be associated with diplopia, nausea, and vomiting. The headache pain is typically frontal and/or occipital in location. Typically the onset of the headache is 12 to 24 hours after a dural puncture and lasts several days if untreated (rarely it can last for months). The other headaches listed rarely have a significant postural component ( Barash: Clinical Anesthesia, ed 7, pp 926-927; Miller: Basics of Anesthesia, ed 6, pp 271-272).

833.What is the CORRECT order of structures (from cephalad to caudad) in the intercostal space? A.Nerve, artery, vein B.Vein, nerve, artery C.Vein, artery, nerve D.Artery, nerve, vein

833. (C)VAN ( Vein, Artery, Nerve) describes the anatomic relationship of the intercostal structures deep to the lower border of the ribs from the cephalad to caudal direction. The block is performed by walking off the inferior edge of the rib with the needle, typically about 5 to 7 cm from midline. The two principal risks are pneumothorax and intravascular injection of local anesthetics. Because of the close proximity of the vein and artery to the nerve, intercostal blocks have relatively high blood levels as compared to other blocks (e.g., epidural, brachial plexus, brachial plexus block, infiltration), and caution with dose is needed if many levels are blocked ( Butterworth: Morgan & Mikhail's Clinical Anesthesiology, ed 5, pp 1018-1019).

834.Which of the following types of regional anesthesia is associated with the GREATEST serum concentration of lidocaine? A.Intercostal B.Epidural C.Brachial plexus D.Femoral nerve block

834. (A)The site of injection of the local anesthetic is one of the most important factors influencing systemic local anesthetic absorption and toxicity. The degree of absorption from the site of injection depends on the blood supply to that site. Areas that have the greatest blood supply have the greatest systemic absorption. For this reason, the greatest plasma concentration of local anesthetic occurs after an intercostal block, followed by caudal epidural, lumbar epidural, brachial plexus, sciatic/femoral nerve block, and subcutaneous ( Barash: Clinical Anesthesia, ed 7, pp 569-570; Miller: Miller's Anesthesia, ed 8, p 1046).

835.Differences in which of the following local anesthetic properties account for the fact that the onset of an epidural block with 3% 2-chloroprocaine is more rapid than 2% lidocaine? A.Protein binding B.pKa C.Lipid solubility D.Concentration

835. (D)Local anesthetics are weak bases. The neutral (non-ionized) form of the molecule is able to pass through the lipid nerve cell membrane, whereas the ionized (protonated) form actually produces anesthesia. Chloroprocaine has the highest pKa of local anesthetics, meaning that a greater percentage of it will exist in the ionized form at any given pH than any of the other local anesthetics. Despite this fact, 3% chloroprocaine has a more rapid onset than 2% lidocaine, presumably because of the greater number of molecules (concentration). However, if one compares onset time for 1.5% lidocaine against 1.5% chloroprocaine, the former will have a more rapid onset ( Miller: Miller's Anesthesia, ed 8, p 1039).

836.A 69-year-old man with a history of diabetes mellitus and chronic renal failure is to undergo placement of a dialysis fistula under regional anesthesia. During needle manipulation for a supraclavicular brachial plexus block, the patient begins to cough and complain of chest pain and shortness of breath. The MOST likely diagnosis is A.Angina B.Pneumothorax C.Phrenic nerve irritation D.Intravascular injection of local anesthetic

836. (B)The risk of pneumothorax is a significant limitation for supraclavicular brachial plexus blocks (traditionally the incidence is 0.5% to 6% depending upon experience; with the ultrasound technique, the incidence may be lower). Furthermore, the technique is difficult to teach and describe. For these reasons, this block should not be performed in patients in whom a pneumothorax or phrenic nerve block (30%-60% of patients) would result in significant dyspnea or respiratory distress. A pneumothorax should be considered if the patient begins to complain of chest pain or shortness of breath or begins to cough during placement of supraclavicular brachial plexus block. In some cases, symptoms of a pneumothorax may be delayed up to 24 hours ( Barash: Clinical Anesthesia, ed 7, pp 961-962; Hebl: Mayo Clinic Atlas of Regional Anesthesia and Ultrasound-Guided Nerve Blockade, ed 1, pp 225-231; Miller: Miller's Anesthesia, ed 8, pp 1727-1728).

837.Each of the following statements is true concerning a femoral nerve block EXCEPT A.The femoral nerve primarily arises from the second to the fourth lumbar nerve roots B.The femoral nerve provides sensation to the anterior and medial aspect of the thigh C.The femoral nerve lies lateral to the femoral artery and femoral vein D.Proper needle placement produces sartorius muscle contraction without patellar movement when electrically stimulated

837. (D)The femoral nerve is the largest branch of the lumbar plexus (it primarily arises from the second to fourth lumbar nerve roots). The femoral nerve divides into an anterior and a posterior division. The anterior division provides motor innervation to the sartorius muscle and cutaneous sensation to the anterior and medial aspects of the thigh. The posterior division innervates the quadriceps muscle and cutaneous sensation to the anterior, medial, and lateral aspects of the knee as well as the articular aspects of the knee joint. The nerve passes under the inguinal ligament and lies just lateral to the femoral artery and vein. If the stimulating needle produces sartorius muscle contraction without patellar movement, then you are too anterior for proper femoral nerve blockade, and the needle needs to be advanced in a more posterior (i.e., deeper) direction. Proper needle placement will elicit quadriceps muscle contraction with patellar elevation that disappears with local anesthetic injection ( Hebl: Mayo Clinic Atlas of Regional Anesthesia and Ultrasound-Guided Nerve Blockade, ed 1, pp 347-362).

838.If a needle is introduced 1.5 cm inferior and 1.5 cm lateral to the pubic tubercle, to which nerve will it lie in close proximity? A.Obturator nerve B.Femoral nerve C.Lateral femoral cutaneous nerve D.Ilioinguinal nerve

838. (A)The obturator nerve provides variable cutaneous innervation of the thigh and can be used to supplement femoral and sciatic nerve blockade for patients having lower extremity surgery. An obturator nerve block is achieved by placement of the needle 1 to 2 cm lateral to and 1 to 2 cm below the pubic tubercle. After contact with the pubic bone, the needle is withdrawn and walked cephalad to identify the obturator canal. Between 10 and 15 mL of local anesthetic should be placed in the canal. If a nerve stimulator is used, contraction of the adductor muscles with nerve stimulation indicates proximity to the nerve ( Barash: Clinical Anesthesia, ed 7, pp 982-983; Hebl: Mayo Clinic Atlas of Regional Anesthesia and Ultrasound-Guided Nerve Blockade, ed 1, pp 386-394; Miller: Miller's Anesthesia, ed 8, pp 1741-1742).

848.Which of the following would hasten the onset and increase the clinical duration of action of a local anesthetic, and provide the GREATEST depth of motor and sensory blockade when used for epidural anesthesia? A.Increasing the volume of local anesthetic B.Increasing the concentration of local anesthetic C.Increasing the dose D.Placing the patient in the head-down position

848. (C)Increasing the total dose (mass) of local anesthetic is more efficacious in hastening the onset and increasing the duration of an epidural anesthetic than increasing the volume or increasing the concentration (while holding the total dose constant) ( Barash: Clinical Anesthesia, ed 7, p 921).

839.The MOST common complication associated with a supraclavicular brachial plexus block is A.Blockade of the phrenic nerve B.Intravascular injection into the vertebral artery C.Blockade of the recurrent laryngeal nerve D.Pneumothorax

839. (A)The most serious complication associated with a supraclavicular brachial plexus block is pneumothorax, which fortunately is rare (0.5%-5%). The most common complication is a phrenic nerve block, which is usually mild and relatively common (30%-60% of blocks). Bilateral supraclavicular blocks, however, are not recommended due to the possibility of bilateral phrenic nerve paralysis or pneumothoraces. Other potential complications include Horner syndrome (ipsilateral eye ptosis, miosis, and anhidrosis), nerve damage or neuritis, infection, or intravascular injection ( Barash: Clinical Anesthesia, ed 7, pp 962; Hebl: Mayo Clinic Atlas of Regional Anesthesia and Ultrasound-Guided Nerve Blockade, ed 1, p 231; Miller: Miller's Anesthesia, ed 8, pp 1727-1728).

840.Which portion of the upper extremity is NOT innervated by the brachial plexus? A.Posterior medial portion of the arm B.Elbow C.Lateral portion of the forearm D.Medial portion of the forearm

840. (A)The arm receives sensory innervation from the brachial plexus except for the shoulder, which is innervated by the supraclavicular nerves from the cervical plexus, and the posterior medial aspect of the arm, which is supplied by the intercostobrachial nerve ( Hebl: Mayo Clinic Atlas of Regional Anesthesia and Ultrasound-Guided Nerve Blockade, ed 1, pp 58-70; Miller: Basics of Anesthesia, ed 6, pp 287-292).

841.Which section of the brachial plexus is blocked with a supraclavicular block? A.Roots/trunks B.Trunks/divisions C.Cords D.Branches

841. (B)The brachial plexus starts out at the root level from the ventral rami of C5-T1 with a small amount from C4 and T2. These roots at the level of the scalene muscle become the three trunks: superior, middle, and inferior. The trunks then divide into the dorsal and ventral divisions at the lateral edge of the first rib. When the divisions enter the axilla, they become the cords: posterior, lateral, and medial. At the lateral border of the pectoralis muscle, they become the five peripheral nerves: radial, musculocutaneous, median, ulnar, and axillary. The interscalene block is at the level of the roots/trunks (but spares the inferior trunk); the supraclavicular block is at the level of the trunks/divisions; the infraclavicular block is at the level of the cords; and the axillary block is at the level of the branches ( Barash: Clinical Anesthesia, ed 7, pp 959-966; Hebl: Mayo Clinic Atlas of Regional Anesthesia and Ultrasound-Guided Nerve Blockade, ed 1, pp 225-226; Miller: Basics of Anesthesia, ed 6, pp 287-292).

842.A celiac plexus block would NOT effectively treat pain resulting from a malignancy involving which of the following organs? A.Uterus B.Stomach C.Pancreas D.Gallbladder

842. (A)The celiac plexus innervates most of the abdominal viscera, including the lower esophagus, stomach, all of the small intestine, and the large intestine up to the splenic flexure as well as the pancreas, liver, biliary tract, spleen, kidneys, adrenal glands, and omentum. The pelvic organs (e.g., uterus, ovaries, prostate, distal colon) are supplied by the hypogastric plexus ( Barash: Clinical Anesthesia, ed 7, pp 1658-1659; Butterworth: Morgan & Mikhail's Clinical Anesthesiology, ed 5, pp 1073-1075).

843.A healthy 27-year-old woman stepped on a nail and is to undergo débridement of a wound on her right great toe. She is anxious about general anesthesia but agrees to an ankle block with mild sedation. Which nerves must be adequately blocked in order to perform the surgery? A.Deep peroneal, posterior tibial, saphenous, sural B.Deep peroneal, saphenous, superficial peroneal, sural C.Deep peroneal, posterior tibial, superficial peroneal, sural D.Deep peroneal, superficial peroneal, posterior tibial, saphenous

843. (D)The great toe is innervated mainly by the deep peroneal, posterior tibial, superficial peroneal, and occasionally by the saphenous nerve. All four of these nerves should be blocked for surgery on the great toe. The sural nerve is the fifth nerve for ankle blocks but covers only the lateral side of the foot, and not the medial side or great toe area ( Butterworth: Morgan & Mikhail's Clinical Anesthesiology, ed 5, pp 1015-1017; Hebl: Mayo Clinic Atlas of Regional Anesthesia and Ultrasound-Guided Nerve Blockade, ed 1, pp 443-452).

844.A 54-year-old man is administered morphine via patient-controlled analgesia (PCA) pump after a left total hip arthroplasty. The pump is programmed to deliver a maximum dose of 2 mg every 15 minutes (lockout time) as needed for patient comfort. The total maximum dose that can be delivered in 4 hours is 30 mg. On the first day the patient receives 15 doses every 4 hours by pressing the delivery button every 15 to 18 minutes. How should his pain control be further managed? A.Discontinue the PCA pump and administer intramuscular morphine B.Increase the lockout time from 15 to 25 minutes C.Change the analgesic from morphine to meperidine D.Increase the dose to 3 mg every 15 minutes as needed up to a total maximum dose of 40 mg every 4 hours

844. (D)Frequent dosing by a patient receiving postoperative analgesia through a PCA pump suggests the need to increase the magnitude of the dose. It is important to keep in mind that a patient should be given a sufficient loading dose of narcotic before initiative therapy with a PCA pump. Otherwise, the patient will be playing the frustrating game of "catch up." The most commonly used narcotics in the United States for PCA pump use are morphine, fentanyl, and hydromorphone. Meperidine should not be used as the narcotic for PCA pumps, since the toxic metabolite normeperidine may accumulate ( Barash: Clinical Anesthesia, ed 7, pp 1626-1627).

845.The mechanism of low-frequency transcutaneous electrical nerve stimulation (TENS) units in relieving pain is A.Direct electrical inhibition of type A-δ and C fibers B.Depletion of neurotransmitter in nociceptors C.Hyperpolarization of spinothalamic tract neurons D.Activation of inhibitory neurons

845. (D)TENS produces a tingling or vibratory sensation in the area in which pads are placed. Although the exact mechanism is unclear, it is thought that TENS units produce analgesia by releasing endogenous endorphins, since its effects are partially blocked by naloxone. These endorphins have an inhibitory effect at the spinal cord level and augment descending inhibitory pathways ( Butterworth: Morgan & Mikhail's Clinical Anesthesiology, ed 5, p 1081; Miller: Miller's Anesthesia, ed 8, pp 2339, 2991).

846.Epidural use of which of the following opioids would result in the GREATEST incidence of delayed respiratory depression? A.Sufentanil B.Fentanyl C.Morphine sulfate D.Hydromorphone

846. (C)Although the more hydrophilic drugs such as morphine have a longer duration of action of analgesia, they also have a higher potential for inducing delayed respiratory depression through cephalad migration in the CNS, as compared with the more lipid-soluble drugs listed in this question ( Barash: Clinical Anesthesia, ed 7, pp 1627-1629; Miller: Miller's Anesthesia, ed 8, p 2983).

849.Select the FALSE statement concerning neurolytic nerve blocks. A.Destruction of peripheral nerves can be followed by a denervation hypersensitivity that is worse than the original pain B.Neurolytic blocks should be reserved for patients with short life expectancies C.Neurolytic blockade with phenol is permanent D.Intrathecal neurolysis may be an effective management for certain pain conditions

849. (C)Alcohol and phenol are similar in their ability to cause nonselective damage to neural tissues. Alcohol causes pain when injected and sometimes is mixed with bupivacaine, whereas phenol is relatively painless. Alcohol has a slightly longer duration of analgesia (3-6 months) compared to phenol (2-3 months). Neural tissue will regenerate; therefore, neurolytic blocks are never "permanent," and neurolysis can lead to denervation hypersensitivity, which can be extremely painful ( Butterworth: Morgan & Mikhail's Clinical Anesthesiology, ed 5, pp 1079-1080; Miller: Miller's Anesthesia, ed 8, p 1911).

850.Transient neurologic symptoms (TNS) after spinal anesthesia is associated with each of the following EXCEPT A.Lidocaine B.Lithotomy position C.Ambulatory anesthesia D.Concentration of local anesthetic injected

850. (D)TNS, previously called transient radicular irritation (TRI), can occur in 4% to 40% of patients after spinal anesthesia with lidocaine in ambulatory patients undergoing surgery in the lithotomy position or knee arthroscopy. The baricity, concentration injected (lidocaine 0.5%-5%), addition of epinephrine, presence of dextrose, or hypotension does not seem to be related to the development of TNS. The symptoms of TNS include pain or sensory abnormalities in the lower back, buttocks, or lower extremities. Although TNS has been reported with all local anesthetics, the incidence is significantly greater with lidocaine ( Barash: Clinical Anesthesia, ed 7, pp 576, 928; Miller: Miller's Anesthesia, ed 8, p 1692).

851.After you select the appropriate ultrasound transducer, you can adjust several factors to optimize the image for regional anesthesia. Which of the following descriptions is FALSE? A.Frequency—higher frequency ultrasound use is better for viewing deep structures B.Depth—adjusted to limit the centimeters of viewing area on the monitor C.Gain—increased gain produces increased brightness D.Frequency—higher frequency ultrasound use produces better image resolution

851. (A)After the proper transducer is selected, you can adjust the frequency, depth, and gain to optimize an image. In general, higher-frequency ultrasound waves provide better image quality (i.e., better resolution) due to the higher number of cycles per second of transmitted and reflected energy used to produce the image. However, higher frequency waves have more signal attenuation at increasing depths and cannot penetrate to deeper tissue levels. Therefore, higher-frequency ultrasound is typically used for shallower structures, and lower frequencies are used for deeper structures. Usually the depth is adjusted so the structure in question is in the center, top-to-bottom, of the image. Increasing the gain increases, or amplifies, the reflected signal energy and increases the brightness of the image ( Hebl: Mayo Clinic Atlas of Regional Anesthesia and Ultrasound-Guided Nerve Blockade, ed 1, pp 99-112).

852.Each of the following is associated with an increased incidence of PDPHs EXCEPT A.Younger adults B.Early ambulation C.Pregnancy D.Large needle size

852. (B)Younger adults have a higher incidence of PDPH than older adults or children. Women have a slightly higher incidence than men. Pregnant women have a higher incidence than nonpregnant women. Since the incidence and severity of PDPH relates to the amount of CSF leakage through the dural hole, it makes sense that the larger the needle and the more holes in the dura, the greater incidence of PDPH. In addition, the shape of the tip of the needle is important; a cutting needle (e.g., Quincke) has a greater incidence of PDPH than noncutting needles (e.g., Whitacre, Sprotte). The incidence of headache has been shown to be less when the dural fibers are split longitudinally rather than when they are cut while the needle is held in a transverse direction. The timing of ambulation relative to dural puncture has not been shown to affect the incidence of postspinal headache. The block should wear off before ambulation is attempted ( Barash: Clinical Anesthesia, ed 7, pp 926-927; Miller: Miller's Anesthesia, ed 8, pp 1694-1695).

853.Each of the following items describes pain in the abdominal viscera EXCEPT A.Pain is transmitted via the vagus nerve B.The nerve fibers are type C C.Pain is characterized by a dull aching or burning sensation D.Distention of the transverse colon causes more pain than surgical transection

853. (A)Virtually all pain arising in the thoracic or abdominal viscera is transmitted via the sympathetic nervous system in unmyelinated type C fibers. Visceral pain is dull, aching, burning, and nonspecific. Visceral pain is caused by any stimulus that excites nociceptive nerve endings in diffuse areas. In this regard, distention of a hollow viscus causes a greater sensation of pain than does the highly localized damage produced by transecting the gut. Although the vagus nerve has a large amount of afferent fibers, they do not include pain fibers ( Brunton: Goodman & Gilman's The Pharmacological Basis of Therapeutics, ed 12, pp 174-175, 567-570; Raj: Practical Management of Pain, ed 3, pp 223-225).

854.Which of the following blocks has the LONGEST duration of action when bupivacaine with epinephrine is administered? A.Axillary B.Epidural C.Infiltration D.Spinal

854. (A)The duration of regional blocks is different between local anesthetics as well as with different location of blocks. When bupivacaine with epinephrine (1:200,000) is used, epidural anesthesia may last 180 to 350 minutes; infiltration anesthesia may last 180 to 240 minutes; and major nerve blocks such as axillary block may last 360 to 720 minutes. Spinal bupivacaine without epinephrine may last 90 to 200 minutes; if epinephrine (0.2-0.3 mg) is added to the spinal block, it will last about 50% longer ( Miller: Miller's Anesthesia, ed 8, pp 1041-1044).

861.A 49-year-old type 1 diabetic patient with a long history of burning pain in the right lower extremity receives a spinal anesthetic with 100 mg of procaine with 5% dextrose. The patient reports no relief in symptoms but has complete bilateral motor blockade. What diagnosis is consistent with this differential blockade examination? A.Diabetic neuropathy B.Central pain C.Myofascial pain D.Complex regional pain syndrome I (RSD)

861. (B)Somatic pain in the extremities is relieved with spinal anesthesia. If a patient fails to obtain pain relief despite complete sympathetic, sensory, and motor blockade, a "central" mechanism for the pain is likely or the lesion causing the pain is higher in the CNS than the level of blockade achieved by the spinal. Central pain states may include encephalization, psychogenic pain, or malingering. Persistence of pain in the lower extremities after successful spinal blockade suggests a central source or psychological source of pain ( Miller: Miller's Anesthesia, ed 8, pp 1898-1910; McMahon: Wall and Melzack's Textbook of Pain, ed 6, Chapter 69).

855.All of the following statements concerning a psoas compartment block are true EXCEPT A.Compartmental block is used to provide unilateral anesthesia to the proximal aspect of the thigh and hip B.Stimulation of the quadriceps muscle demonstrates good needle placement C.Complete leg anesthesia can be obtained when combined with a sciatic nerve block D.Continuous catheters are not used because the amount of drug infused would lead to toxicity

855. (D)Psoas compartment block is also called the posterior lumbar plexus block and can be used for any procedure in which a lumbar plexus block is required, but most often it is used for analgesia for the proximal aspect of the thigh and hip. When combined with a sciatic block, complete leg anesthesia will result. Remembering that the femoral nerve (which innervates the quadriceps muscles) is a distal branch helps one to understand why quadriceps muscle contraction is useful in locating the plexus with a stimulating needle (1-1.5 mA). If the hamstring muscles are stimulated, the needle is too caudally located, and the needle should be aimed in a more cephalad direction. Continuous psoas catheters are commonly used for postoperative analgesia ( Barash: Clinical Anesthesia, ed 7, pp 978-980; Hebl: Mayo Clinic Atlas of Regional Anesthesia and Ultrasound-Guided Nerve Blockade, ed 1, pp 333-345).

856.A 35-year-old woman receives a popliteal block for ankle and foot surgery. Which other nerve must be blocked in order to have complete anesthesia of the foot? A.Superficial peroneal nerve B.Sural nerve C.Saphenous nerve D.Posterior tibial nerve

856. (C)All of the nerves of the foot (with the exception of the saphenous) are derived from the sciatic nerve. The sciatic nerve distally becomes the tibial and peroneal nerves, which can be blocked at the popliteal fossa for surgery below the knee. The saphenous nerve is a branch of the femoral nerve and provides sensory innervation along the medial aspect of the lower leg between the knee and the medial malleolus, and must also be blocked for surgery below the knee ( Hebl: Mayo Clinic Atlas of Regional Anesthesia and Ultrasound-Guided Nerve Blockade, ed 1, pp 423-426; Butterworth: Morgan & Mikhail's Clinical Anesthesiology, ed 5, pp 1013-1015).

857.The most common complication of a celiac plexus block is A.Hypotension B.Seizure C.Retroperitoneal hematoma D.Constipation

857. (A)The sympathectomy produced by a celiac plexus block causes hypotension by decreasing preload to the heart. This complication can be avoided by volume loading the patient with lactated Ringer solution. By blocking the sympathetic chain, unopposed parasympathetic activity may also result in increased gastrointestinal activity and transient diarrhea. Back pain is also common. Paraplegia may result from spasm of the lumbar segmental arteries that perfuse the spinal cord, direct vascular or neurologic injury, or retrograde spread of drug to the nerve roots and spinal cord. Seizure is possible with an intravascular injection. Retroperitoneal hematoma is also possible, but rare ( Barash: Clinical Anesthesia, ed 7, pp 1658-1659; Butterworth: Morgan & Mikhail's Clinical Anesthesiology, ed 5, pp 1073-1074).

858.The occipital portion of the skull receives sensory innervation from A.Spinal accessory nerve (nerve XI) B.Facial nerve (nerve VII) C.Ophthalmic branch of trigeminal nerve (nerve V) D.Cervical plexus

858. (D)The occiput receives sensory innervation from the greater and lesser occipital nerves (C2 and C3 spinal roots), which are terminal branches of the cervical plexus. Blockade of these nerves is usually carried out as a diagnostic step in the evaluation of head and neck pain ( Barash: Clinical Anesthesia, ed 7, pp 946-947, 958-959; Butterworth: Morgan & Mikhail's Clinical Anesthesiology, ed 5, p 1065).

859.Each of the following is a potential complication of thoracic paravertebral blocks EXCEPT A.Pneumothorax B.Epidural spread of local anesthetic C.Hypertension D.Total spinal

859. (C)Thoracic paravertebral blocks are used for surgical anesthesia and postoperative analgesia for breast, axillary, or chest wall surgery. The major complication is a pneumothorax. Since the paravertebral space is continuous with the epidural space medially, epidural spread may result if large volumes of local anesthetic are injected into the paravertebral. Typically 5 mL are injected at each of three sites for unilateral paravertebral blocks, and 3 mL per each of six sites (three on each side) if bilateral paravertebral blocks are performed. If the needle is directed too medially, then the intrathecal space may be entered (dural sleeves extend to the level of the intervertebral foramina) with the possibility of a total spinal if 5 to 10 mL is injected. The sympathetic chain is in the anterior part of the paravertebral space, and sympathetic blockade may develop; however, hypotension would be more likely than hypertension to develop from blocking the sympathetic chain ( Barash: Clinical Anesthesia, ed 7, pp 972-975; Butterworth: Morgan & Mikhail's Clinical Anesthesiology, ed 5, pp 1019, 1067-1068; Hebl: Mayo Clinic Atlas of Regional Anesthesia and Ultrasound Guided Peripheral Nerve Blockade, ed 1, pp 323-329).

860.After placement of an epidural catheter in a 55-year-old patient for total hip arthroplasty, an entire epidural dose is administered into the subarachnoid space. Physiologic effects consistent with subarachnoid injection of large volumes of local anesthetic include all of the following EXCEPT A.Hypotension and bradycardia B.Respiratory depression C.Constricted pupils D.Possible cauda equina syndrome

860. (C)With the unintentional injection of an epidural dose of local anesthetic into the subarachnoid space, spinal anesthesia develops rapidly. Blockade of the sympathetic fibers (T1-L2) produces hypotension, particularly if the patient is hypovolemic. Bradycardia is produced by blocking the cardiac accelerator fibers (T1-T4). Respiratory arrest is due to hypoperfusion of the respiratory centers as well as paralysis of the phrenic nerve (C3-C5). The pupils become dilated (mydriasis) after intrathecal injection of large quantities of local anesthetics; they will return to normal size after the block recedes. Cauda equina syndrome has occasionally developed when the epidural dose was unintentionally administered into the subarachnoid space (most commonly with chloroprocaine). If one suspects an unintentional placement of the epidural dose subarachnoid, supportive methods are initially used (the basic ABC's of resuscitation). One can also aspirate CSF from the epidural catheter (if it was inserted) to help remove some of the drug as well as reducing the pressure in the subarachnoid space, which might help better perfuse the spinal cord and decrease the chance of cauda equina syndrome developing ( Barash: Clinical Anesthesia, ed 7, pp 927-928; Miller: Miller's Anesthesia, ed 8, pp 1690, 1702; Southorn: Reducing the potential morbidity of an unintentional spinal anaesthetic by aspirating cerebrospinal fluid, Br J Anaesth 76:467-469, 1996).

862.An 18-year-old man has a seizure during placement of an interscalene brachial plexus block with 0.5% bupivacaine. The anesthesiologist begins to hyperventilate the patient's lungs with 100% O 2 using an anesthesia bag and mask. The rationale for this therapy includes all of the following EXCEPT A.The therapy helps to prevent and treat hypoxia B.Hyperventilation decreases blood flow and delivery of local anesthetic to the brain C.Hyperventilation elevates the seizure threshold D.Hyperventilation induces alkalosis and converts local anesthetics to the protonated (ionized) form, which is less likely to cross the cell membranes

862. (D)During a seizure, both arterial hypoxemia and acidosis (metabolic and respiratory) develop due to the increased oxygen consumption from contracting muscles and hypoventilation that occurs. Administration of 100% O 2 helps to prevent and treat hypoxemia. Elevated CO 2 not only enhances cerebral blood flow and delivery of local anesthetic to the brain but also diffuses into neural tissue, causing intracellular pH to fall. Because local anesthetics are either amino esters or amino amides, lowering the pH allows more binding of hydrogen ions to the amino group, making it more ionic or protonated, which traps the local anesthetic inside the cells. Hyperventilation can reverse many of the changes that occur with acidosis (i.e., causes cerebral vasoconstriction and can decrease delivery of local anesthetic to the brain). Hyperventilation induces hypokalemia and respiratory alkalosis, both of which result in hyperpolarization of nerve membranes and elevation of the seizure threshold. Hyperventilation also raises the patient's pH (respiratory alkalosis) and converts local anesthetics into the non-ionized (nonprotonated) form, which crosses the membrane more easily than the ionized form, which is detrimental. Benzodiazepines and/or propofol are used to suppress the seizure activity ( Barash: Clinical Anesthesia, ed 7, p 575; Miller: Miller's Anesthesia, ed 8, pp 1048-1050).

863.Para-aminobenzoic acid is a metabolite of A.Mepivacaine B.Ropivacaine C.Bupivacaine D.Procaine

863. (D)Para-aminobenzoic acid is a metabolite of the ester-type local anesthetics. Local anesthetics may be placed into two distinct categories based on their chemical structure: amino esters or amino amides. The amides (two i's in the name), which are ropivacaine, lidocaine, etidocaine, prilocaine, mepivacaine, and bupivacaine, are metabolized in the liver. The ester local anesthetics (one i in the name) are cocaine, procaine, chloroprocaine, tetracaine, and benzocaine. These drugs are metabolized by the enzyme pseudocholinesterase found in the blood. Para-aminobenzoic acid is a metabolic breakdown product of ester anesthetic and is responsible for allergic reactions in some individuals ( Butterworth: Morgan & Mikhail's Clinical Anesthesiology, ed 5, pp 270-271; Hemmings: Pharmacology and Physiology for Anesthesia, ed 1, pp 298-303).

864.Which statement concerning peripheral nerve structure and function is FALSE? A.Both nonmyelinated and myelinated nerves are surrounded by Schwann cells B.The speed of propagation of an action potential along a nerve axon is greatly enhanced by myelin C.Generation of an action potential is an "all-or-nothing" phenomenon D.Myelination renders nerves less sensitive to local anesthetic blockade

864. (D)Peripheral nerve axons are always enveloped by a Schwann cell. The myelinated nerves may be enveloped many times by the same Schwann cell. Transmission of nerve impulses (i.e., action potentials) along nonmyelinated nerves occurs in a continuous fashion, whereas transmission along myelinated nerves occurs by saltatory conduction from one node of Ranvier to the next. Myelination speeds transmission of neurologic impulses; it also renders nerves more susceptible to local anesthetic blockade. An action potential is associated with an inward flux of sodium that occurs after a certain membrane threshold has been exceeded ( Miller: Miller's Anesthesia, ed 8, pp 1031-1035).

865.A 42-year-old woman with a morbid fear of general anesthesia receives an interscalene block for shoulder arthroscopy consisting of 20 mL 0.5% ropivacaine. Much of her arm, shoulder, and hand are numb, but the patient complains of pain as the incision is made at the upper portion of the shoulder. The most appropriate next step is to A.Repeat block B.Perform intercostobrachial block C.Perform superficial cervical plexus block D.Perform a deep cervical plexus block

865. (C)The needle insertion site for an interscalene block is C6 (i.e., lateral to the cricoid cartilage). Local anesthetics usually spread to C5, C6, and C7, which supply much, but not all, of the cutaneous innervation to the shoulder. With low-to-moderate volume blocks, there will be sparing of the C3-C4 nerve roots, which supply some of the innervation to the anterior shoulder. Of note, C8 and T1 may also be spared, often resulting in the need for ulnar nerve supplementation if this block were used for a hand operation. Complete anesthesia for shoulder arthroscopy may require a supplemental superficial cervical plexus with use of low-to-moderate volumes of a local anesthetic ( Hebl: Mayo Clinic Atlas of Regional Anesthesia and Ultrasound Guided Peripheral Nerve Blockade, ed 1, pp 185-193).

866.According to the 2004 American Society of Regional Anesthesia and Pain Medicine (ASRA) practice advisory on infectious complications of regional anesthesia and pain medicine, the MOST important action to maintain aseptic technique and prevent cross-contamination during regional anesthesia techniques is A.Wearing a surgical gown B.Hand washing C.Using soap and water instead of alcohol-based antiseptics D.Using povidone-iodine (e.g., Betadine) instead of alcohol-based chlorhexidine to scrub

866. (B)Hand washing is one of the most important techniques to prevent infections, especially when alcohol-based antiseptic solutions are used with sterile gloves. Although soap and water remove bacteria, they do not effectively kill organisms. Antiseptic solutions with alcohol appear to be better than nonalcoholic antiseptics (e.g., povidone-iodine). Nail length does not appear to be a risk factor for infections, because the majority of bacterial growth occurs along the proximal 1 mm of nail adjacent to the subungual skin. Universal use of gowns and gloves does not appear to be better than gloves alone in preventing infections in intensive care units (ICUs) and presumably is less important than adequate hand washing and use of sterile gloves ( Hebl: Infectious complications: a new practice advisory, Reg Anesth Pain Med 31:289-290, 2006; Hebl: The importance and implications of aseptic techniques during regional anesthesia, Reg Anesth Pain Med 31:311-323, 2006).

874.Discharge criteria from the postanesthesia care unit would be reached FASTEST after a 20- to 30-mL volume of which of the following epidurally administered local anesthetics? A.3% 2-Chloroprocaine B.2% Lidocaine C.0.75% Ropivacaine D.0.5% Levobupivacaine

874. (A)Procaine and 2-chloroprocaine have a short duration of action; lidocaine, mepivacaine, and prilocaine have an intermediate duration of action; and etidocaine, bupivacaine, levobupivacaine, tetracaine, and ropivacaine have a long duration of action. For similar sensory anesthesia, a higher concentration of local anesthetic is needed for the short duration of local anesthetics compared with both the intermediate and long-duration agents, because they are less potent ( Barash: Clinical Anesthesia, ed 7, pp 920-922; Miller: Miller's Anesthesia, ed 8, pp 1710-1711).

867.A 75-year-old woman with a history of pulmonary embolism is scheduled for a right lower lobectomy for lung cancer. She is receiving dalteparin (Fragmin) for deep vein thrombosis (DVT) prophylaxis. How long after her last dose should one wait prior to placement of a thoracic epidural? A.12 hours B.24 hours C.72 hours D.No waiting is necessary since the dose for prophylaxis is low

867. (A)In patients taking low-molecular-weight heparin (LMWH) (e.g., enoxaparin, dalteparin, tinzaparin), caution should be exercised before proceeding with an epidural or spinal anesthetic because of the risk of producing an epidural or spinal hematoma. The amount of time between the last dose of the LMWH and the relative safety of starting a central neuraxial block depends on the dose of the LMWH. At the lower doses, used for thromboprophylaxis, the LMWH should be held at least 10 to 12 hours prior to the block. At the higher doses, used to treat an established DVT, one should wait at least 24 hours after the last dose of LMWH prior to the block ( Barash: Clinical Anesthesia, ed 7, p 929; Miller: Miller's Anesthesia, ed 8, pp 1702, 2344-2345; Horlocker: Regional anesthesia in the patient receiving antithrombotic or thrombolytic therapy: American Society of Regional Anesthesia and Pain Medicine Evidence-Based Guidelines (Third Edition), Reg Anesth Pain Med 35:64-101, 2010).

868.How long should a patient be off clopidogrel (Plavix) before a central neuraxial block is performed? A.24 hours B.7 days C.14 days D.No waiting necessary

868. (B)Taking nonsteroidal anti-inflammatory drugs (NSAIDs), ticlopidine, and clopidogrel exert effects on platelet function. NSAIDs are not a problem if given alone before epidural or spinal anesthesia; however, patients taking ticlopidine should wait 14 days and patients taking clopidogrel should wait 7 days before having a neuraxial block placed, because of the increased risk of spinal hematoma formation. Keep in mind that caution is always needed and that the ASRA statement "Careful preoperative assessment of the patient to identify alterations of health that might contribute to bleeding is crucial" is important ( Barash: Clinical Anesthesia, ed 7, p 929; Horlocker: Regional anesthesia in the patient receiving antithrombotic or thrombolytic therapy: American Society of Regional Anesthesia and Pain Medicine Evidence-Based Guidelines (Third Edition), Reg Anesth Pain Med 35:64-101, 2010).

869.Addition of bicarbonate to local anesthetics results in A.Delayed onset of action B.Reduced toxicity C.Increased duration of action D.Reduced pain with skin infiltration

869. (D)Adding sodium bicarbonate to local anesthetic solutions hastens the onset of action of the local anesthetics, especially when the local anesthetic solution contains epinephrine (which is produced at a lower pH). By raising the pH, more of the local anesthetic is in the non-ionized, more lipid-soluble state. Raising the pH too much (i.e., >6.05-8) would cause precipitation of the local anesthetic. Some studies have shown that alkalization of the local anesthetic may decrease the duration of a peripheral block, especially if epinephrine was not added. It also seems to decrease pain with skin infiltration. Pain on injection can also be decreased by a slow injection of the local anesthetic ( Barash: Clinical Anesthesia, ed 7, pp 567-568; Miller: Miller's Anesthesia, ed 8, p 1040).

870.Through which of the following would a spinal needle NOT pass during a midline placement of a subarachnoid block in the L3-L4 lumbar space? A.Supraspinous ligament B.Interspinous ligament C.Posterior longitudinal ligament D.Dura mater

870. (C)This figure shows the anatomic structures that must be traversed by the spinal needle during the performance of a subarachnoid block. The structures include the skin, subcutaneous tissue, supraspinous ligament, interspinous ligament, ligamentum flavum, the epidural space, and finally the dura (posteriorly). If you were to continue to advance the spinal needle, you would encounter the dura (anteriorly) while exiting the subarachnoid space, the posterior longitudinal ligament, the periosteum of the vertebral body, and finally bone ( Cousins: Neural Blockade in Clinical Anesthesia and Management of Pain, ed 3, p 205).

871.What epidural dose of bupivacaine will give sensory analgesia similar to 10 mL of 2% lidocaine? A.5 mL of 0.25% B.10 mL of 0.25% C.5 mL of 0.5% D.10 mL of 0.5%

871. (D)In the epidural space, bupivacaine (as well as levobupivacaine) is four times more potent than lidocaine, so 0.5% bupivacaine is similar to 2% lidocaine for analgesia. The duration of the bupivacaine block will be longer because bupivacaine has a long duration of action and lidocaine has an intermediate duration of action. In addition, motor block would be less for bupivacaine compared with lidocaine, since there is more of a greater difference between sensory and motor block for bupivacaine as compared with lidocaine ( Barash: Clinical Anesthesia, ed 7, pp 920-922; Miller: Basics of Anesthesia, ed 6, pp 134, 277).

872.Each of the following additives to a spinal anesthetic possesses analgesic properties EXCEPT A.Clonidine B.Hydromorphone C.Epinephrine D.All of the above have analgesic properties

872. (D)Drugs with α-adrenergic agonist activity (phenylephrine, 2-5 mg; epinephrine, 0.2-0.5 mg; clonidine, 75-150 mg) possess some analgesic activity but less than narcotics and local anesthetics. In addition, these intrathecal α-adrenergic agonists may reduce systemic/vascular uptake of local anesthetics, thereby enhancing their effects, including hypotension. Clonidine alone, when administered neuraxially, is an effective analgesic. Neostigmine has some mild analgesia properties, but experience is limited. Opioids (e.g., fentanyl, sufentanil, hydromorphone, and morphine) added to the spinal solution enhance surgical anesthesia and provide postoperative pain relief. Fentanyl or sufentanil is commonly added for short surgical procedures (outpatient), whereas hydromorphone or morphine can be used when longer postoperative analgesia is desired for inpatients ( Barash: Clinical Anesthesia, ed 7, pp 919-920; Miller: Miller's Anesthesia, ed 8, pp 1693, 2983).

873.Which of the following local anesthetics is inappropriately paired with a clinical application because of its properties or toxicity? A.Tetracaine, topical anesthesia B.Bupivacaine, intravenous anesthesia C.Prilocaine, infiltrative anesthesia D.Chloroprocaine, epidural anesthesia

873. (B)For topical anesthesia, lidocaine, tetracaine, cocaine, dibucaine, and benzocaine are effective, as well as the combination of lidocaine and prilocaine, or EMLA cream. For intravenous regional anesthesia or Bier blocks, many drugs have been used. Ester local anesthetics are not used for IV regional blocks because they can be broken down in the bloodstream (by plasma ester hydrolysis), which can shorten the drug's duration of action and can also cause thrombophlebitis of the vein (reported with chloroprocaine). Because cardiovascular collapse has been reported with bupivacaine, it should not be used for intravenous regional anesthesia. Lidocaine and prilocaine are used for Bier blocks because of their relative safety. For infiltrative and epidural anesthesia, almost all local anesthetics can be used (with the exception of cocaine and benzocaine, which are used only topically) ( Butterworth: Morgan & Mikhail's Clinical Anesthesiology, ed 5, p 272; Miller: Miller's Anesthesia, ed 8, pp 1041-1044, 1736).

875.A caudal block (performed under sevoflurane general anesthesia) with 0.25% bupivacaine and 1:200,000 epinephrine is planned for postoperative analgesia after bilateral inguinal hernia repair in a 5-month-old patient. Each of the following would be consistent with an intravascular injection EXCEPT A.Systolic blood pressure increase by greater than 15 mm Hg B.Heart rate decrease by greater than 10 beats/min C.Ventricular extrasystoles D.Increase in T-wave amplitude >25% over baseline

875. (B)Under sevoflurane general anesthesia, an increase in the T-wave amplitude of 25% (usually in lead II), an increase in heart rate of 10 beats/min, or a systolic blood pressure increase greater than 15 mm Hg is considered a positive dose response to an epinephrine-containing local anesthetic solution. Under total intravenous anesthesia, an increase in blood pressure is more sensitive than an increase in T-wave amplitude or an increase in heart rate. As always, slow incremental dosing is safer than a large bolus dose ( Barash: Clinical Anesthesia, ed 7, pp 1247-1248; Davis: Smith's Anesthesia for Infants and Children, ed 8, pp 456-457).

876.Which is NOT a potential complication of a stellate ganglion block? A.Recurrent laryngeal nerve paralysis B.Subarachnoid block C.Brachial plexus block D.Increased heart rate

876. (D)All of the choices listed are potential complications of stellate ganglion blockade except an increase in heart rate. The stellate ganglion supplies sympathetic fibers to the upper extremity and head and some to the heart. Loss of the cardiac acceleratory fibers may slow the heart rate, not speed it up. Other potential complications of stellate ganglion blockade include accidental injection of the local anesthetic into a vertebral artery, resulting in seizure, phrenic nerve paralysis, and inadvertent cervical epidural ( Miller: Basics of Anesthesia, ed 6, pp 707-710; Miller: Miller's Anesthesia, ed 8, p 1732).

877.An axillary block using the transarterial approach with 0.5% bupivacaine and epinephrine (1:200,000) is performed in a 70-kg patient. A 30-mL quantity is injected posterior to the axillary artery and 30 mL anterior to it. How many milligrams have been injected, and was the maximum recommended dose exceeded? A.150 mg bupivacaine, 150 μg epinephrine did not exceed maximum dose B.150 mg bupivacaine, 150 μg epinephrine exceeded maximum dose C.300 mg bupivacaine, 300 μg epinephrine did not exceed maximum dose D.300 mg bupivacaine, 300 μg epinephrine exceeded maximum dose

877. (D)A total of 60 mL of 0.5% bupivacaine with epinephrine (1:200,000) was used. A 0.5% solution = 0.5 g in 100 mL of fluid = 500 mg/100 mL = 5 mg/mL. A 1:200,000 solution means 1 g in 200,000 mL = 1000 mg/200,000 mL = 1 mg/200 mL = 1000 μg/200 mL = 5 μg/mL. Therefore 60 mL of 0.5% bupivacaine contains 60 mL × 5 mg/mL = 300 mg bupivacaine and 1:200,000 epinephrine 60 mL × 5 μg/mL = 300 μg of epinephrine. For a major nerve block, the maximum recommended dose with epinephrine (1:200,000) is 500 mg for lidocaine and mepivacaine, 600 mg with prilocaine, and 225 mg with bupivacaine. Epinephrine is used in the local anesthetic to check for intravascular injection of the incremental doses and is not contraindicated but should be included for this block. Typically 40 to 45 mL is used for the transarterial approach to the axillary block ( Barash: Clinical Anesthesia, ed 7, p 572; Miller: Miller's Anesthesia, ed 8, pp 1043, 1728-1729).

878.Three days after knee arthroscopy under spinal anesthesia, a 55-year-old patient complains of double vision and difficulty hearing. The other likely finding would be A.Headache B.Fever C.Weakness in legs D.Mental status changes

878. (A)Postdural puncture headaches (spinal headaches) usually develop within 12 to 72 hours after a dural puncture but may develop immediately or take months to develop. The most characteristic symptom is a postural component in which the headache occurs in the upright position and is usually completely gone when the patient is in the supine position. The headache is typically frontal and/or occipital in location. Other symptoms include nausea, vomiting, anorexia, visual disturbances (blurred vision, double vision, photophobia), and occasionally hearing loss (routinely found with auditory testing) ( Butterworth: Morgan & Mikhail's Clinical Anesthesiology, ed 5, pp 969-970; Miller: Basics of Anesthesia, ed 6, pp 271-272).

879.Which of the following statements is TRUE concerning transversus abdominis plane (TAP) block? A.Ultrasound is useful in finding the intercostal nerves B.The local anesthetic is injected directly into the transversus abdominis muscle C.The subcostal, ilioinguinal, and iliohypogastric nerves are blocked D.10 mL of local anesthetic is all that is needed for good spread

879. (C)TAP block is used to provide abdominal wall analgesia. The subcostal (T12), ilioinguinal (L1), and iliohypogastric (L1) nerves are the nerves primarily blocked. Ultrasound is often used to locate the proper plane where the local anesthetic is injected, since the nerves are too small to visualize. After visualization of the three abdominal wall muscles, the external oblique, the internal oblique, and the transversus abdominis muscles, the needle is inserted. The local anesthetic is injected into the muscle plane between the internal oblique and the transversus abdominis muscles (which is where these nerves travel) and not the muscle for effective analgesia. Typically 20 to 30 mL of local anesthetic (e.g., 2 mg/kg of bupivacaine) is needed for adequate spread of local anesthetic ( Barash: Clinical Anesthesia, ed 7, pp 975-976; Butterworth: Morgan & Mikhail's Clinical Anesthesiology, ed 5, pp 1021-1022).

880.Which of the following nerves can be electrically stimulated at the ankle to produce flexion of the toes? A.Posterior tibial nerve B.Saphenous nerve C.Deep peroneal nerve D.Superficial peroneal nerve

880. (A)Five nerves are blocked when performing an ankle block. The saphenous, superficial peroneal, and sural nerves are all sensory below the ankle, and electrical stimulation would have no effect. Stimulation of the posterior tibial nerve causes flexion of the toes by stimulating the flexor digitorum brevis muscles and abduction of the first toe by stimulating the abductor hallucis muscles. The posterior tibial nerve also is sensory to most of the plantar part of the foot. Stimulation of the deep peroneal nerve causes extension of the toes by stimulating the extensor digitorum brevis muscles. The deep peroneal nerve has a small sensory branch for the first interdigital cleft. From a practical standpoint, many anesthesiologists perform a purely infiltration block of these nerves. If a nerve stimulator is used, it is mainly used to find the posterior tibial nerve, which can be difficult to anesthetize if small volumes of local anesthetic are administered. The posterior tibial nerve can be difficult to stimulate in diabetic patients with diabetic neuropathy ( Hebl: Mayo Clinic Atlas of Regional Anesthesia and Ultrasound-Guided Nerve Blockade, ed 1, pp 443-446; Barash: Clinical Anesthesia, ed 7, pp 990-992).

881.Which motor response from peripheral nerve stimulation is INCORRECTLY paired with the appropriate nerve? A.Musculocutaneous nerve—flexion of the forearm at the elbow B.Radial nerve—extension of all digits as well as the wrist and forearm C.Ulnar nerve—abduction of the thumb D.Median nerve—flexion of the wrist, pronation of the forearm

881. (C)Peripheral nerve stimulation is a common technique when performing axillary nerve blocks. The desired motor response from the nerve can be seen with 0.5 mA or less. The musculocutaneous nerve elicits elbow flexion. The radial nerve elicits extension of all the digits, the wrist, and the elbow, as well as supination of the forearm. The ulnar nerve elicits flexion at the wrist, fourth and fifth digits, and adduction (not abduction) of the thumb. The median nerve elicits flexion at the wrist and second and third digits as well as opposition of the thumb and pronation of the forearm ( Butterworth: Morgan & Mikhail's Clinical Anesthesiology, ed 5, p 992; Hebl: Mayo Clinic Atlas of Regional Anesthesia and Ultrasound-Guided Nerve Blockade, ed 1, pp 256-260).

882.During an airway exam, a 53-year-old patient mentions that his right thumb tingles and then becomes numb if he extends his head for more than a few seconds. This symptom MOST likely represents a(n) A.Unstable C-spine B.Lhermitte's phenomenon C.C6 nerve root irritation D.C8 radiculopathy

882. (C)Unilateral numbness or paresthesia in the upper extremity during extension of the neck usually represents nerve root impingement at the vertebral foramina. C6 nerve distribution is the thumb. Specifically, unilateral degenerative changes restrict the foramen to such a degree that it compresses and irritates the nerve root traversing the vertebral foramen when the head is extended. Treatment ranges from NSAIDs to steroids and may require surgical intervention if there is muscle weakness. Lhermitte sign, named after Jean Lhermitte, occurs when head flexion causes shooting sensations down the back and into the lower limbs. It is a sign of posterior column disease ( Hebl: Mayo Clinic Atlas of Regional Anesthesia and Ultrasound-Guided Nerve Blockade, ed 1, p 86; Miller: Miller's Anesthesia, ed 8, p 1725).

883.When performing an interscalene block with a peripheral nerve stimulator, you note diaphragmatic movement. You should now A.Inject the local anesthetic, as the needle is in an appropriate location B.Redirect the needle in an anterior direction C.Redirect the needle in a posterior direction D.Advance the needle about 0.5 cm more and inject

883. (C)Although a successful interscalene block causes ipsilateral phrenic nerve paralysis in almost 100% of patients, identifying the phrenic nerve means that you are anterior to the brachial plexus and that you should reposition your needle. You should redirect the needle in a posterior direction ( Barash: Clinical Anesthesia, ed 7, pp 959-961; Hebl: Mayo Clinic Atlas of Regional Anesthesia and Ultrasound-Guided Nerve Blockade, ed 1, pp 195-199; Miller: Miller's Anesthesia, ed 8, pp 1725-1727).

884.During placement of an interscalene block, the patient becomes hypotensive, bradycardic, apneic, and cyanotic. The MOST likely cause is A.Vertebral artery injection B.Phrenic nerve blockade C.Total spinal D.Stellate ganglion block

884. (C)With an intravascular injection, the main symptoms would most likely be CNS toxicity (e.g., seizures), as blood flow is directly to the brain. The Bezold-Jarisch reflex (hypotension and bradycardia) has been reported in awake, sitting patients undergoing shoulder surgery with an interscalene block. This may be related to intracardiac mechanoreceptors being stimulated by the decreased venous return in the sitting position. This leads to decreased sympathetic tone and increased parasympathetic tone. Breathing is still present with this reflex. Block of the stellate ganglion would produce Horner syndrome, which is not associated with breathing abnormalities. Injection into the intrathecal space is uncommon, but possible (especially if the needle is not pointed in the caudal direction), and would lead to a total spinal block with little local anesthetic injected (e.g., hypotension, bradycardia respiratory paralysis that would lead to cyanosis) ( Barash: Clinical Anesthesia, ed 7, pp 959-961; Hebl: Mayo Clinic Atlas of Regional Anesthesia and Ultrasound-Guided Nerve Blockade, ed 1, pp 203-205; Miller: Miller's Anesthesia, ed 8, pp 1725-1727).

885.The reason that ropivacaine is marketed as pure S enantiomers is because the S form is associated with A.Increased potency B.Longer duration C.Reduced cardiac toxicity D.Reduced incidence of anaphylaxis

885. (C)The pipecoloxylidide local anesthetics (mepivacaine, bupivacaine, ropivacaine, and levobupivacaine) are chiral drugs, which means that they have an asymmetric carbon atom (i.e., have a left or S and a right or R hand configuration). Mepivacaine and bupivacaine are produced as racemic mixtures (50% S:50% R). The pure S forms show reduced neurotoxicity and reduced cardiotoxicity (e.g., ropivacaine and levobupivacaine). Clinical studies suggest that the pure S forms have a slight decrease in potency and a shorter duration of action compared with racemic mixtures. Lidocaine is an achiral compound (i.e., has no chiral carbon atom) ( Barash: Clinical Anesthesia, ed 7, pp 566-567; Brunton: Goodman & Gilman's The Pharmacological Basis of Therapeutics ed 12, pp 565-574).

886.Nerves that originate from the sacral plexus include each of the following EXCEPT A.Femoral nerve B.Tibial nerve C.Sciatic nerve D.Common peroneal nerve

886. (A)Nerves to the lower extremity emerge from the L1-S4 nerve roots. The upper roots (mainly L1-L4) form the lumbar plexus, which gives rise to the genitofemoral (L1-L2), lateral femoral cutaneous (L2-L3), obturator (L2-L4), and the femoral (L2-L4) nerves. A branch from the lumbar plexus (L4) along with the sacral plexus (L4-S3) gives rise to the sciatic nerve. Branches of the sciatic nerve include the common peroneal (branches to make the superficial and deep) and the tibial, and the sural nerves ( Barash: Clinical Anesthesia, ed 7, pp 952-955; Miller: Miller's Anesthesia, ed 8, p 1736).

887.The only technique shown to prevent anesthetic-related nerve injury during placement of peripheral nerve blocks is A.Ultrasound-guided regional technique B.Transarterial technique C.Nerve stimulator D.None of the above

887. (D)Anesthetic-related nerve injuries to the brachial plexus are rare and poorly understood. The only way to minimize nerve injury is to minimize trauma to neural fibers. Although ultrasound-guided technique is promising, currently there is no clinical evidence for this ( Neal: Upper extremity regional anesthesia: Essentials of our current understanding, 2008, Reg Anesth Pain Med 34:134-170, 2009).


Conjuntos de estudio relacionados

Biology Exam 2: All Mastering Questions

View Set

Microeconomics Exam #3 (Chapters 6,7, and 8)

View Set

QuizzesAccident investigation has traditionally focused on preventing accidents by concentrating on simple theories to determine "what happened," "who was responsible," and "when did it occur."

View Set

Ch. 33 Cerebrovascular Disorders

View Set

ECON 2301 Midterm Review (Ch. 1-7, 17)

View Set

AP Gov Unit 3 Civil Liberties and Civil Rights Main Ideas

View Set

8 objections to Mills Utilitarianism

View Set

Urinary System Lecture Objectives

View Set

Chapter 22 Lymphatic System and Immunity Anatomy 2

View Set